Please I need help!!!!!

Please I Need Help!!!!!

Answers

Answer 1

Answer:

learn how to do it

Step-by-step explanation:

lol

Answer 2

Answer:

<J = 60.26

<L = 28.84

JL = [tex]2\sqrt{65}[/tex]

Step-by-step explanation:

One is given that triangle (JKL) is a right triangle. The measurements for the legs (JK) and (KL) are given; (8) and (14) respectively. One can use the Pythagorean theorem to solve for the unknown side, the hypotenuse. The Pythagorean theorem states the following,

[tex]a^2+b^2=c^2[/tex]

Where (a) and (b) are the legs or sides adjacent to the right angle of the right triangle. (c) is the hypotenuse or the side opposite the right angle of the right triangle. Substitute in the given values and solve for the unknown,

[tex]a^2+b^2=c^2\\\\8^2+14^2=c^2\\\\64+196=c^2\\\\260=c^2\\\\2\sqrt{65}=c[/tex]

Now one has to find the unknown angles. This can be done using the right triangle trigonometric ratios. These ratios are used to describe the relationship between the sides and an angle of a triangle. These ratios are as follows,

[tex]sin(\theta)=\frac{opposite}{hypotenuse}\\\\cos(\theta)=\frac{adjacent}{hypotenuse}\\\\tan(\theta)=\frac{opposite}{adajcent}[/tex]

Bear in mind that the sides are named relative to the angle one is describing it from. Thus, the name of the (opposite) and (adjacent) sides change relative to the angle of description. However, the hypotenuse is the side opposite the right angle, this side never changes its name.

Solve for (<J)

Since one has the measures of all of the sides, one can use any ratio. In this case, let's use the ratio of tangent (tan).

[tex]tan(\theta)=\frac{opposite}{adjacent}\\\\\tan(\theta)=\frac{14}{8}\\\\\theta=tan^-1(\frac{14}{8})\\\\\theta=60.256[/tex]

Solve for (<L)

The sum of angles in any triangle is (180) degrees. One can use this property here. Form an equation by adding up all of the angle measures, and solve for (<L)

(<L) + (<J) + (<K) = 180

(<L) + 60.26 + 90 = 180

(<L) + 151.16 = 180

(<L) = 28.84


Related Questions

Which expression is equivalent to (x^1/2 y^-1/4 z)^-2 ?

Answers

Answer:

[tex] {( {x}^{ \frac{1}{2} }. {y}^{ - \frac{1}{4} } .z) }^{ - 2} \\ = (x. {y}^{ - \frac{1}{2} } . {z}^{ - 2} )[/tex]

y^(1/2) / (xz^2 hope this helps :)

\Suppose that a person has an average heart rate of 88.6 beats/min. (Express your answers to problems in this section to the correct number of significant figures and proper units.) (a) How many beats does he or she have in 5.0 y

Answers

Answer:

He or she has 232,840,800 beats in five minutes.

Step-by-step explanation:

This question is solved by proportions.

In this question:

A year has 365 days.

Each day has 24 hours.

Each hour has 60 minutes.

Minutes in five years:

5*365*24*60 = 2,628,000 minutes.

How many beats in five minutes?

88.6 beats/min, 2,628,000 minutes. So the number of bears is:

88.6*2628000 = 232,840,800

He or she has 232,840,800 beats in five minutes.

Find the measurement of angle b.

Answers

Answer:

b = 55

Step-by-step explanation:

The two angles add to 90 degrees

b+35 = 90

Subtract 35 from each side

b +35-35 = 90-35

b = 55

Answer:

90-35=55

Answer is D

Step-by-step explanation:

As evident, the line is bisecting the right angle. One side of the bisection is given as 35. Thus by deduction:

90-35=b

90-35=55

55=b

3. TELEPHONE NUMBERS What is the probability that a 7-digit telephone number with the digits 5, 1, 6, 5, 2, 1 , and 5 is the number 550 - 5211 ?

Answers

There are about 10 billion possible phone numbers so the chances are about 2 in a billion

Write about a time when you had an unexpected expense. How did you deal with the unexpected expense? What are some unexpected expenses you might face in the next five years?

Answers

Answer:

Some unexpected expenses I have had were Pet Emergencies with my dog

or Major Auto Repairs if my car gets in an accident or home repairs if my child writes on the wall or breaks a whole in it. I dealt with them by selling stuff that I dont use to pay it off. Some expenses I might face in the next 5 years would be Medical emergencies for my family just incase anything were to happen to them

Find the equation of the line with slope m=1/4 that contains the point (4,5).

Answers

Answer: x - 4y = -16

Step-by-step explanation: Since we're given a point and a slope, we can write the equation of this line using the point-slope formula.

y  -  y₁  =  m(x  -  x₁)

The given point, (4, 5), represents (x₁, y₁).

So if we plug all our given information into the formula,

including our slope of 1/4, we get y - 5 = 1/4(x - 4).

To put our equation in standard form, first, distribute

the 1/4 through the parentheses.

So we get y - 5 = 1/4x - 1.

Remember that standard form cannot have any fractions in it so our next step is to multiply both sides of the equation by 4 to get rid of the fraction.

That gives us 4y - 20 = x - 4.

Now moves the -20 to the right side by adding 20 to both sides.

So we get 4y = x + 16.

Now subtract x from both sides to get -x + 4y = 16.

Finally, our coefficient on our x term must be positive so we

divide both sides by -1 to get x - 4y = -16.

Let V1=(1,2,-5)^T v2=((2,5,6)^T and b=(7,4,-3)^T cn be expressed as a linear combination of V1 and V2?

Answers

I assume the question is whether the vector b is a linear combination of v₁ and v₂. If it is, then that means there are scalars c₁ and c₂ such that

b = c₁ v₁ + c₂ v₂   ==>   (7, 4, -3) = c₁ (1, 2, -5) + c₂ (2, 5, 6)

This gives the system of equations,

c₁ + 2c₂ = 7

2c₁ + 5c₂ = 4

-5c₁ + 6c₂ = -3

which has no solution, since

-2 (c₁ + 2c₂) + (2c₁ + 5c₂) = -2(7) + 4   ==>   c₂ = -10

but substituting this into any two equations in the system gives contradictory results. For instance,

c₁ + 2c₂ = c₁ - 20 = 7   ==>   c₁ = 27

2c₁ + 5c₂ = 2c₁ - 50 = 4   ==>   c₁ = 27

-5c₁ + 6c₂ = -5c₁ - 60 = -3   ==>   c₁ = -57/5

This means that b is not a linear combination of v₁ and v₂.

A car travels 32 km due north and
then 46 km in a direction 40° west of
north. Find the DIRECTION of the car's
resultant vector.

Answers

Answer:

Ifa,bandcare the

sides opposite to anglesA,B,&C.

By Cosine Rule,

b  

2

=32  

2

+46  

2

−(2×32×46cos140°)

b  

2

=1024+2116−2944cos140°

b  

2

=3140−2944cos140°

b  

2

=3140+2255.234841

b  

2

=5395.234841

b=73.45

By Sine Rule,

sinC

32

=  

sin140°

73.45

 

sinC=  

73.45

32sin140°

 

sinC=0.2800

C=16.26°

The direction of the resultant

vector is measured from0°east

to the resultant, this angle isθ−90°,

90°is the total angle in

the first quadrant.

∴θ−90°=C+ϕ

ϕ=50°{Alternate angles are equal}

∴The direction of the car’s

resultant vector is50°+16.26°=66.26°south of east

Step-by-step explanation:

what digit could be in the ten millions place of a number that is greater than 70,000,000 but less than 100,000,000

Answers

Answer:

hiiiiiiiiiiiiiiiiiiiiiiiiiiii bro

My car payment is 312.67. My water bill is 12percent of the car payment. How much is the water bill

Answers

Answer:

275.15

Step-by-step explanation:

312.67 * 12% = 37.52

312.67 - 37.52 = $275.15

|-2 + 5 | - |3 - 12| - 13=

Answers

Answer:

-19

3 - 9 - 13 = -19

Step-by-step explanation:

Which calculation correctly uses prime factorization to write V48 in simplest form?
A. 48 = V2 -2 -2 -2 - 3 = 2V12
B. 48 = 14 - 12 = 2/12
C. 48 = V2 -2 -2 -2 - 3 = 4V3
D. V48 = 16 - 3 = 4V3​

Answers

Answer:

C

[tex] \sqrt{48} = \sqrt{2 \times 2 \times 2 \times 2 \times 3} \\ = \sqrt{16 \times 3} \\ = 4 \sqrt{3} [/tex]

The prime factorization to write [tex]\sqrt{48}[/tex] in the simplest form as  

48 = V2 -2 -2 -2 - 3 = 4V3

What is Prime Factorization in Math?

Prime factorization of any number indicates to express that number as a product of prime numbers. A prime number exists as a number that has precisely two factors, 1 and the number itself.

We require to express 48 as the product of its prime factors as 48.

= 2 [tex]\times[/tex] 3 [tex]\times[/tex] 2 [tex]\times[/tex] 2 [tex]\times[/tex] 2.

Therefore, [tex]$\sqrt{48}=\sqrt{ (2 \times 2 \times 2 \times 2 \times 2)}[/tex]

[tex]$=4 \sqrt{3}$[/tex]

[tex]$4 \sqrt{3}$[/tex] exists in the lowest radical form.

Therefore, the correct answer is option C. 48 = V2 -2 -2 -2 - 3 = 4V3.

To learn more about Prime Factorization

https://brainly.com/question/24612438

#SPJ2

Choose the graphs that indicate equations with no solution

Answers

Answer:

A and E

Step-by-step explanation:

Given

Graphs A to E

Required

Which do not have solutions

When there are no point of intersection between lines and/or curves of a graph, then such graph has no solution.

Using the above description as a yard stick, the first (A) and the last (E) graph have no solution.

The Pythagorean Theorem (image below)

Answers

3 sqr 2! hope this helps

Helpppp quiccckkkkkm

Answers

Answer:

a) G

b) C

Step-by-step explanation:

Triangle A is congruent to triangle G

Triangle D is congruent to Triangle C

Terri wants to know the value of n in the equation 7.638xn=763.8 what value for n makes the equation true?

Answers

Answer: 100

Step-by-step explanation:

Since we are given the equation 7.638 × n = 763.8 and told that Terri wants to know the value of n in the equation that will make it true. To do this goes thus:

7.638 × n = 763.8

7.638n = 763.8

Divide both side by 7.638

7.638n/7.638 = 763.8/7.638

n = 100

Therefore, n = 100 will make the equation true.

If tickets for a concert cost $9 each, how many tickets can you buy if you have $135?
Write an equation and solve by guess-and-check.
9
c. 9x = 135; 15 tickets
135, 15 tickets
a
X
b. 9 + x = 135; 15 tickets
d. 9 - x = 135: 15 tickets

Answers

Answer:

15

Step-by-step explanation:

135 divided by 9 = 15

what's nine plus ten

Answers

Answer:

21

Step-by-step explanation:

2Kewl4U!but the actual answer is 19

Answer:

Hi, it's 19

Step-by-step explanation:

9+10=19   or    10+9=19

Marsha deposited $10,000 into a savings account 5 years ago. The simple interest rate is 3%.
much money did Marsha earn in interest

Answers

Answer:

She earned 1500 in interest

Step-by-step explanation:

10,000 × 0.03 × 5 = 1500

Your welcome :)

In 1992, the moose population in a park was measured to be 4600. By 1998, the population was measured again to be 4240. If the population continues to change linearly

1.Find a formula for the moose population, P , in terms of t, the years since 1990.

2.What does your model predict the moose population to be in 2009?

Answers

Answer:

P(t) = - 60t + 4720

3580

Step-by-step explanation:

Given :

m = (y2 - y1)/(x2 - x1)

m = (4250 - 4600) / (8 - 2)

m = - 360 / 6

m = - 60

Using the point slope relation :

y - y1 = m(x - x1)

y - 4600 = - 60(x - 2)

y - 4600 = - 60x + 120

y - 4600 + 4600 = - 60x + 120 + 4600

y = - 60x + 4720

P(t) = - 60t + 4720

To predict population in 2009 :

2009 - 1990 = 19 years

Put t = 19

P(19) = - 60(19) + 4720

P(19) = - 1140 + 4720

P(19) = 3580

Determine whether the following statement is always, sometimes, or never true.

If two pairs of corresponding sides of two right triangles are congruent, then the triangles are
congruent.​

Answers

Answer:

this statement is true because then the triangles will be congruent by rhs axiom

Anne can pack 84 boxes in 6 minutes. How many boxes can she pack per minute?​

Answers

Answer:14 boxes per minute.

Step-by-step explanation:

This is a simple equation.

84/6 = 14 boxes per minute.

Malik collects rare stamps and has a total of 212 stamps. He has 34 more domestic stamps than foreign stamps. Let x represent the number of domestic stamps and let y represent the number of foreign stamps.
How many foreign stamps does Malik have?

Answers

The answer that represents the total number of stamps that Malik has collected is: x + y = 212. The equation that represents the difference in the number of foreign and domestic stamps Malik collected is: x - y = 34. The System of Linear Equations that represent the situation is: x - y = 34 and x + y = 212.

Answer:

answer in picture
c, b, d

Step-by-step explanation:

Find the balance of the account earning interest compound annually.
$7000 at 4% for 2 years

Answers

9514 1404 393

Answer:

  $7571.20

Step-by-step explanation:

Each year, the balance is multiplied by 1+4% = 1.04. After 2 years, the balance is ...

  $7000 × 1.04 × 1.04 = $7571.20

_____

Additional comment

The repeated multiplication is usually indicated with an exponent:

  $7000×1.04²

write the first ten prime numbers in the line. delete 6 digits to get the largest possible number

Answers

9514 1404 393

Answer:

  7317192329

Step-by-step explanation:

The first 10 primes are ...

  2, 3, 5, 7, 11, 13, 17, 19, 23, 29

The number of interest will keep getting larger as long as the first (or next needed) digit keeps getting larger. That is, we want to delete any digit that is not greater than the one previous. Following this rule until we have deleted 6 digits, we find the largest number to be ...

  7317192329

_____

The digits deleted are those in bold in the list of primes.

Answer:

7317192329

Step-by-step explanation:

hope it helps

The mass of a dime is about 0.002268 kg. Which is the best estimate of the mass?
A 2 x 10-3
B 2 x 10-2
C 2 x 102
D 2 x 103

Answers

Answer:

A.

[tex] = > \: 2 \times {10}^{ - 3} [/tex]

if f(x)=3x-2 and g(x)=2x+1, find (f-g)(x)

A. x-3
B. 5x-3
C. 5x-1
D. 3-x​

Answers

Answer:

A. x-3

Step-by-step explanation:

f(x)=3x-2 and g(x)=2x+1

(f-g)(x) = 3x-2  - (2x+1)

           Distribute the minus sign

(f-g)(x) = 3x-2  - 2x-1

       Combine like terms

           = x -3

Cual es la tercera parte de la mitad de 3/4

Answers

Answer:

traducir mi respuesta al español si es necesario

1/8

Step-by-step explanation:

lets start with the basics 1/2 of 3/4 to do it we multiply the fractions

1        3         3

    x  

2        4           8

the answer to that problem was 3/8 if u still dont understand multiplying fractions you can look it up and put on spanish subtitles

now to find 1/3 of 3/8 we do the same thing

1       3

    x

3       8        

we get 3/24 we can make the number smaller by reducing it. you can look up a video on reducing

jada makes sparkiling juice by mixing 2 cups of sparkiling water with every 3 cups of aplle juice.

how much sparkiling water does jada need if she uses 15 cups of apple juice?
solve the answer with a full explanation

Answers

Answer:

10 cups

Step-by-step explanation:

we can set up a proportion of cups of water over cups of apple juice

2/3 = w/15

if you cross-multiply you will get:

3w = 30

w = 30/3

w = 10

COS I = sin 34°
What is the value of x?

Please help me or I will fail this class

Answers

Not sure what we are solving, please eloborate on what you need solved, for now I can’t answer
I’m confused can u say what needs to be solved maybe then I can help
Other Questions
6a4+5b3 when a=3 and b=3 What political events set the stage for the Salem Witch Trials? Simplify 4528-12+42Please show working Why did american join World War one? You are a bidder in an independent private values auction, and you value the object at $4,000. Each bidder perceives that valuations are uniformly distributed between $1,500 and $9,000. Determine your optimal bidding strategy in a first-price, sealed-bid auction when the total number of bidders (including you) is: I am sorry_______ last nighta.to disturb youb.for disturbing youc.to have disturbed youd.for having disturbed you A letter to my uncle why I left my former school three reasons Woolly mammoths are an extinct species of elephant that lived in very cold climates.They had very small ears.Compared with the large ears of modern elephants, small ears would have assisted insurvival of the mammoths byA. Decreasing the core body temperatureB. Increasing the effects of heat conductionC. Increasing the radiation of heat from the bodyD. Decreasing the surface area through which heat is lost Qu se puede decir acerca de la relacin entre el ser humano y el mediogeogrfico se puede establecer? Simplify (-4a^5b) (2a^2b) Tengo un pasaporte colombiano en la mano y estoy por pasar la aduana conmis padres, que tambin tienen pasaportes colombianos. Los pasaportesson - help me please lol this is an important grade what is this 8.000 fl oz = ml Jacinta received a scholarship from the american meteorological society and was able to pay 43% of one year of her college with the scholarship money the remainder 11, 033.75 for the year was posted for with a student loan what was the total cost of 4 years In what two southern states didthe black population outnumberthe white population? Need help with figuring out this zingoes problem asap, willing to give brainliest! is it a physical or chemical change when a candle is lit Helpppp me with explanation also please:((((( 15. Set up a proportion and use it to solve for x. Show your work.210X3 GUYS PLS HELP ME!!! which is a sign of alcohol overdose?a)- normal speech b)- heavy snoring c)- mild fever d)- slow, irregular breathingthank you!!!